Đến nội dung

Hình ảnh

Kỹ thuật sử dụng bdt Cauchy-Schwart trong tích phân


  • Please log in to reply
Chủ đề này có 13 trả lời

#1
tranquocluat_ht

tranquocluat_ht

    Thượng sĩ

  • Thành viên
  • 235 Bài viết
Mình thấy kỹ năng này tạo ra lời giải ngắn gọn cho nhiều bài toán BĐT tích phân. Trong topic này mình đưa ra một số bài, hi vọng tìm được cơ sở và những dự đoán cần thiết để chứng minh một lớp BĐT tích phân. Xin bắt đầu với một ví dụ đơn giản.
Ví dụ 1: Cho hàm $f: [0;1] \to R$ thỏa mãn $\int_0^1{f(x)}dx=\int_0^1{xf(x)}dx=1$. Chứng minh rằng

$\int_0^1f^2(x)dx \ge 4.$


Lời giải: Áp dụng BĐT Cauchy-Schwarz ta có:

$(\int_0^1(x+k)f(x)dx)^2 \le \int_0^1(x+k)^2dx. \int_0^1{f^2(x)dx}$

.

Suy ra

$\int_0^1f^2(x)dx \ge \frac{(k+1)^2}{k^2+k+\frac{1}{3}}=f(k)$

.

Đến đây chỉ việc chọn $k$ sao cho $f(k)$ đạt giá trị lớn nhất.
Tìm được $k=4$.
Bài toán được giải quyết trọn vẹn.

Mời các bạn tiếp tục với bài tiếp theo.

Ví dụ 2: Cho hàm $f: [0;1] \to R$ thỏa mãn $\int_0^1{xf(x)}dx=0$. Chứng minh rằng

$\int_0^1f^2(x)dx\geq 4(\int_0^1f(x)dx)^2$.


PS: Vấn đề đặt ra là nhừng BĐT dạng nào, có dấu hiệu gì thì ta sử dụng phương pháp này. Mong nhận được ý kiến từ các bạn.


@ a Luật!: Mạn phép a Luật, em xin sửa cái tiêu đề thành cái topic cho a em thảo luận nhé!
Anh em thảo luận về BĐT tích phân ở đây nhé!
Lưu ý: Các bài spam mình sẽ xóa và không thông báo!

Bài viết đã được chỉnh sửa nội dung bởi khacduongpro_165: 23-01-2013 - 21:54


#2
tranquocluat_ht

tranquocluat_ht

    Thượng sĩ

  • Thành viên
  • 235 Bài viết
Ví dụ 3: Cho hàm $f: [0;1] \to R$ liên tục. Chứng minh rằng

$\int_0^1f(x)dx. \int_0^1x^4f(x)dx \geq \frac{4}{15}\int_0^1f^2(x)dx$.



#3
phudinhgioihan

phudinhgioihan

    PĐGH$\Leftrightarrow$TDST

  • Biên tập viên
  • 348 Bài viết

Ví dụ 2: Cho hàm $f: [0;1] \to R$ thỏa mãn $\int_0^1{xf(x)}dx=0$. Chứng minh rằng

$\int_0^1f^2(x)dx\geq 4(\int_0^1f(x)dx)^2$.


PS: Vấn đề đặt ra là nhừng BĐT dạng nào, có dấu hiệu gì thì ta sử dụng phương pháp này. Mong nhận được ý kiến từ các bạn.


Về khoản "dạng nào" thì mình không khẳng định, bởi lẽ cũng cùng là cái áo đó, cái quần đó nhưng có người mặc đẹp, người mặc không hợp..nói chung tùy khả năng sáng tạo và cách sử dụng của từng người mà mỗi người đúc kết một cái riêng cho mình.

Do $\int_0^1{xf(x)}dx=0$ nên $\int_0^1 f(x)dx =\frac{1}{b} \int_0^1 f(x) (ax+b) dx\;\;, (a,b) \in \mathbb{R}_*^2 $

Do đó, sử dụng bdt C-S :

$$(\frac{1}{b} \int_0^1 f(x) (ax+b))^2 dx \le \frac{1}{b^2}( \int_0^1 (ax+b)^2dx \int_0^1 f^2(x)dx) $$

Ta chọn $a,b$ sao cho $\int_0^1 (ax+b)^2 dx=\frac{1}{4b^2} $

Tức $a,b$ thỏa mãn : $a^3+3ab(a+b)=\frac{3a}{4b^2}$

Chọn $b=1 $, dễ tìm được $a=\frac{-3}{2} $

Tức ta có đánh giá :

$$(\int_0^1 f(x)dx )^2=(\int_0^1 f(x)(1-\frac{3}{2}x) dx)^2 \le \int_0^1 (1-\frac{3}{2}x)^2dx \int_0^1 f^2(x)dx$$

$$\le \frac{1}{4} \int_0^1 f^2(x)dx $$

từ đây có đpcm.

Phủ định của giới hạn Hình đã gửi

Đó duy sáng tạo ! Hình đã gửi


https://phudinhgioihan.wordpress.com/

#4
duong vi tuan

duong vi tuan

    Thượng sĩ

  • Thành viên
  • 229 Bài viết

Ví dụ 3: Cho hàm $f: [0;1] \to R$ liên tục. Chứng minh rằng

$\int_0^1f(x)dx. \int_0^1x^4f(x)dx \geq \frac{4}{15}\int_0^1f^2(x)dx$.

Lấy f(x) =1 . thif Vt= 1/5 < Vp= 4/15 .
NGU
Hình đã gửi

#5
viet 1846

viet 1846

    Gà con

  • Thành viên
  • 224 Bài viết

Lấy f(x) =1 . thif Vt= 1/5 < Vp= 4/15 .


Mình không hiểu bạn định nói gì ở đây??

Ví dụ 4: Cho $f$ là hàm khả vi liên tục đến cấp 2 trên đoạn $[0;2]$ thỏa mãn $f(0)-2f(1)+f(2)=1$ chứng minh rằng:

\[\int\limits_0^2 {{{\left( {f''\left( x \right)} \right)}^2}} dx \ge \frac{3}{2}\]

Bài viết đã được chỉnh sửa nội dung bởi Hoàng Quốc việt: 23-01-2013 - 17:45


#6
duong vi tuan

duong vi tuan

    Thượng sĩ

  • Thành viên
  • 229 Bài viết

Mình không hiểu bạn định nói gì ở đây??


à mình chọn hàm f(x) =1 thì thấy chiều bdt ngược ???

Bài viết đã được chỉnh sửa nội dung bởi duong vi tuan: 24-01-2013 - 09:49

NGU
Hình đã gửi

#7
phudinhgioihan

phudinhgioihan

    PĐGH$\Leftrightarrow$TDST

  • Biên tập viên
  • 348 Bài viết

Ví dụ 3: Cho hàm $f: [0;1] \to R$ liên tục. Chứng minh rằng

$\int_0^1f(x)dx. \int_0^1x^4f(x)dx \geq \frac{4}{15}\int_0^1f^2(x)dx$.


Bất đẳng thức phải là chiều ngược lại, tức
$\int_0^1f(x)dx. \int_0^1x^4f(x)dx \le \frac{4}{15}\int_0^1f^2(x)dx$

Với $h$ làm một hàm liên tục trên $[0;1]$

$$\int_0^1 h^2(x)dx \int_0^1 f^2(x)dx \ge \left(\int_0^1 h(x)f(x) dx \right)^2 $$

Ta chọn $h$ sao cho có $k \in \mathbb{R}_+$ để $$\left(\int_0^1 h(x)f(x) dx \right)^2 \ge k \int_0^1 f(x)dx \int_0^1 x^4 f(x)dx $$

Chọn $h(x)=ax^4+b$

Suy ra $$\left( a\int_0^1 x^4f(x)dx+b\int_0^1 f(x)dx \right)^2 \ge k \int_0^1 f(x)dx \int_0^1 x^4 f(x)dx$$

$$VT \ge 4ab \int_0^1 f(x)dx \int_0^1 x^4 f(x)dx$$

Vậy ta cần chọn $a;b$ sao cho

$$\int_0^1 (ax^4+b)^2 dx \int_0^1 f^2(x)dx \ge 4ab \int_0^1 f(x)dx \int_0^1 x^4 f(x)dx $$

Chọn $a,b$ không đồng thời bằng 0 sao cho

$$\int_0^1 f(x)dx \int_0^1 x^4 f(x)dx \le \dfrac{5a^2+18ab+45b^2}{180ab}\int_0^1 f^2(x)dx$$

Chọn $a=1,b=\frac{1}{3}$ và ta có ngay đpcm.

Ta cũng có $\min_{(a,b) \in {\mathbb{R}^*_+}^2 } \dfrac{5a^2+18ab+45b^2}{180ab}=\dfrac{4}{15}$, do đó ta luôn có

$$\forall k\ge \dfrac{4}{15}, \int_0^1 f(x)dx \int_0^1 x^4 f(x)dx \le k \int_0^1 f^2(x)dx $$

Bài viết đã được chỉnh sửa nội dung bởi phudinhgioihan: 24-01-2013 - 11:40

Phủ định của giới hạn Hình đã gửi

Đó duy sáng tạo ! Hình đã gửi


https://phudinhgioihan.wordpress.com/

#8
phudinhgioihan

phudinhgioihan

    PĐGH$\Leftrightarrow$TDST

  • Biên tập viên
  • 348 Bài viết

Mình không hiểu bạn định nói gì ở đây??

Ví dụ 4: Cho $f$ là hàm khả vi liên tục đến cấp 2 trên đoạn $[0;2]$ thỏa mãn $f(0)-2f(1)+f(2)=1$ chứng minh rằng:

\[\int\limits_0^2 {{{\left( {f''\left( x \right)} \right)}^2}} dx \ge \frac{3}{2}\]


Bài này ảo nhỉ :))

Sử dụng khai triển Taylor với phần dư tích phân

$$f(x)=f(1)+f'(1)(x-1)+\int_1^x f''(t)(x-t)dt$$

$$\Rightarrow f(0)=f(1)-f'(1)+\int_0^1 tf''(t)dt \;\;, f(2)=f(1)+f'(1)+\int_1^2 f''(t)(2-t)dt$$

$$\Rightarrow f(0)+f(2)=2f(1)+\int_0^1 tf''(t)dt+\int_1^2(2-t)f''(t)dt$$

$$\Rightarrow 1 \le \sqrt{\int_0^1 t^2 dt \int_0^1 [f''(t)]^2 dt}+\sqrt{\int_1^2 (2-t)^2dt \int_1^2 [f''(t)]^2dt}$$

$$\Leftrightarrow 1 \le \dfrac{1}{\sqrt{3}} \sqrt{\int_0^1 [f''(x)]^2dx}+\dfrac{1}{\sqrt{3}} \sqrt{\int_1^2 [f''(x)]^2dx}$$

$$\Rightarrow \sqrt{3} \le \sqrt{2(\int_0^1 [f''(x)]^2dx+\int_1^2 [f''(x)]^2dx)}$$

$$\Leftrightarrow \int_0^2 [f''(x)]^2dx \ge \dfrac{3}{2}$$

Hic, cực nhọc quá :(

Phủ định của giới hạn Hình đã gửi

Đó duy sáng tạo ! Hình đã gửi


https://phudinhgioihan.wordpress.com/

#9
phudinhgioihan

phudinhgioihan

    PĐGH$\Leftrightarrow$TDST

  • Biên tập viên
  • 348 Bài viết
Ví dụ 5: Sáng tạo bất đẳng thức.

Cho $[a;b] \subset \mathbb{R}$, các hàm được xét đều liên tục trên $[a;b]$.

Ta có: $$\left(\int_a^b h(x)f(x)dx \right)^2 \le \int_a^b h^2(x)dx \int_a^b f^2(x)dx $$

Chọn $h(x)=m g(x)+n \int_a^b g(x)dx$

$$\Rightarrow \left( m\int_a^bf(x)g(x)dx+n\int_a^b g(x)\int_a^b f(x)dx \right)^2 \le \int_a^b f^2(x)dx \left(m^2 \int_a^b g^2(x)dx+(2mn+(b-a)n^2)\left(\int_a^b g(x)dx \right)^2 \right) $$

Chọn $f$ là hàm đã xác định, đặt $p=\int_a^b f(x)dx \;\;, q=\int_a^b f^2(x)dx $

$$\Rightarrow m^2 \left(\int_a^b f(x)g(x)dx \right)^2+(n^2p^2-q(2mn+(b-a)n^2)) \left( \int_a^b g(x)dx \right)^2+2mnp \int_a^b g(x)dx \int_a^b f(x)g(x)dx \\ \le m^2q\int_a^b g^2(x)dx$$

Để đơn giản, chọn $a=-b$ và $f$ là hàm lẻ , khi đó $p=\int_{-b}^b f(x)dx=0$

$$\Rightarrow m^2 \left(\int_{-b}^b f(x)g(x)dx \right)^2 \le m^2q \int_{-b}^b g^2(x)dx+2qn(m+n) \left( \int_{-b}^b g(x)dx \right)^2 $$

Chọn $f(x)=x$ và $m \neq 0$

$$\Rightarrow \left(\int_{-b}^b xg(x)dx \right)^2 \le \dfrac{2b^3}{3}\left( \int_{-b}^b g^2(x)dx+\dfrac{2n(m+n)}{m^2} \left( \int_{-b}^b g(x)dx \right)^2 \right)$$

Với $b=1, n=\dfrac{-1}{2},m=1$ thì ta có một đề thi (của nước nào đó quên rồi :)) )

$$\int_{-1}^1 g^2(x)dx \ge \dfrac{1}{2}\left( \int_{-1}^1 g(x)dx \right)^2 +\dfrac{3}{2} \left(\int_{-1}^1 xg(x) dx\right)^2$$


Chọn $m=n$ và ta có bất đẳng thức khá đẹp

$$\left(\int_{-b}^b xg(x)dx \right)^2 \le \dfrac{2b^3}{3}\left( \int_{-b}^b g^2(x)dx +4\left( \int_{-b}^b g(x)dx \right)^2 \right) $$

Từ bất đẳng thức tổng quát trên ta có thể chế ra nhiều bất đẳng thức đẹp (có gai nữa ) khác mà nhìn bề ngoài rất khó sử dụng C-S.

Phủ định của giới hạn Hình đã gửi

Đó duy sáng tạo ! Hình đã gửi


https://phudinhgioihan.wordpress.com/

#10
viet 1846

viet 1846

    Gà con

  • Thành viên
  • 224 Bài viết

Bài này ảo nhỉ :))

Sử dụng khai triển Taylor với phần dư tích phân

$$f(x)=f(1)+f'(1)(x-1)+\int_1^x f''(t)(x-t)dt$$

$$\Rightarrow f(0)=f(1)-f'(1)+\int_0^1 tf''(t)dt \;\;, f(2)=f(1)+f'(1)+\int_1^2 f''(t)(2-t)dt$$

$$\Rightarrow f(0)+f(2)=2f(1)+\int_0^1 tf''(t)dt+\int_1^2(2-t)f''(t)dt$$

$$\Rightarrow 1 \le \sqrt{\int_0^1 t^2 dt \int_0^1 [f''(t)]^2 dt}+\sqrt{\int_1^2 (2-t)^2dt \int_1^2 [f''(t)]^2dt}$$

$$\Leftrightarrow 1 \le \dfrac{1}{\sqrt{3}} \sqrt{\int_0^1 [f''(x)]^2dx}+\dfrac{1}{\sqrt{3}} \sqrt{\int_1^2 [f''(x)]^2dx}$$

$$\Rightarrow \sqrt{3} \le \sqrt{2(\int_0^1 [f''(x)]^2dx+\int_1^2 [f''(x)]^2dx)}$$

$$\Leftrightarrow \int_0^2 [f''(x)]^2dx \ge \dfrac{3}{2}$$

Hic, cực nhọc quá :(


:D cách của em.


Theo BDT $Cauchy-Schwarz$ ta có: (Tây cũng có)

\[\int\limits_0^1 {{x^2}dx} .\int\limits_0^1 {{{\left( {f''\left( x \right)} \right)}^2}} dx \ge {\left( {\int\limits_0^1 {xf''\left( x \right)dx} } \right)^2}\]

Mà \[\int\limits_0^1 {xf''\left( x \right)dx} = \left. {xf'\left( x \right)} \right|_0^1 - \int\limits_0^1 {f'\left( x \right)} dx = f'\left( 1 \right) - f\left( 1 \right) + f\left( 0 \right)\]

Nên suy ra:

\[\frac{1}{3}\int\limits_0^1 {{{\left( {f''\left( x \right)} \right)}^2}} dx \ge {\left( {f'\left( 1 \right) - f\left( 1 \right) + f\left( 0 \right)} \right)^2}\]


Lại theo $Cauchy-Schwarz$

\[\int\limits_1^2 {{{\left( {x - 2} \right)}^2}dx} .\int\limits_1^2 {{{\left( {f''\left( x \right)} \right)}^2}} dx \ge {\left( {\int\limits_1^2 {\left( {x - 2} \right)f''\left( x \right)dx} } \right)^2}\]

mà \[\int\limits_1^2 {\left( {x - 2} \right)f''\left( x \right)dx} = \left. {\left( {x - 2} \right)f'\left( x \right)} \right|_1^2 - \int\limits_1^2 {f'\left( x \right)dx} = f'\left( 1 \right) - f\left( 2 \right) + f\left( 1 \right)\]

Nên:

\[\frac{1}{3}\int\limits_1^2 {{{\left( {f''\left( x \right)} \right)}^2}dx} \ge {\left( { - f'\left( 1 \right) + f\left( 2 \right) - f\left( 1 \right)} \right)^2}\]

Suy ra:

\[\frac{1}{3}\int\limits_0^2 {{{\left( {f''\left( x \right)} \right)}^2}} dx \ge {\left( { - f'\left( 1 \right) + f\left( 2 \right) - f\left( 1 \right)} \right)^2} + {\left( {f'\left( 1 \right) - f\left( 1 \right) + f\left( 0 \right)} \right)^2} \ge \frac{1}{2}{\left( { - f'\left( 1 \right) + f\left( 2 \right) - f\left( 1 \right) + f'\left( 1 \right) - f\left( 1 \right) + f\left( 0 \right)} \right)^2} = \frac{1}{2}\]

Ta có ĐPCM. :D

Bài viết đã được chỉnh sửa nội dung bởi Hoàng Quốc việt: 24-01-2013 - 17:01


#11
viet 1846

viet 1846

    Gà con

  • Thành viên
  • 224 Bài viết
Ví dụ 6: Cho hàm liên tục $f: [-1,1]\to R$ chứng minh.



$$\int\limits_{ - 1}^1 {{f^2}(x)dx} \ge \frac{1}{2}{\left( {\int\limits_{ - 1}^1 {f(x)dx} } \right)^2} + \frac{3}{2}{\left( {\int\limits_{ - 1}^1 {xf(x)dx} } \right)^2}.$$

Bài viết đã được chỉnh sửa nội dung bởi Hoàng Quốc việt: 24-01-2013 - 17:53


#12
phudinhgioihan

phudinhgioihan

    PĐGH$\Leftrightarrow$TDST

  • Biên tập viên
  • 348 Bài viết

Ví dụ 6: Cho hàm liên tục $f: [-1,1]\to R$ chứng minh.



$$\int\limits_{ - 1}^1 {{f^2}(x)dx} \ge \frac{1}{2}{\left( {\int\limits_{ - 1}^1 {f(x)dx} } \right)^2} + \frac{3}{2}{\left( {\int\limits_{ - 1}^1 {xf(x)dx} } \right)^2}.$$


Ví dụ 5: Sáng tạo bất đẳng thức.


Với $b=1, n=\dfrac{-1}{2},m=1$ thì ta có một đề thi (của nước nào đó quên rồi :)) )

$$\int_{-1}^1 g^2(x)dx \ge \dfrac{1}{2}\left( \int_{-1}^1 g(x)dx \right)^2 +\dfrac{3}{2} \left(\int_{-1}^1 xg(x) dx\right)^2$$


Đã giải ngay trên kia rồi mà :))

Phủ định của giới hạn Hình đã gửi

Đó duy sáng tạo ! Hình đã gửi


https://phudinhgioihan.wordpress.com/

#13
dangnamneu

dangnamneu

    Hạ sĩ

  • Thành viên
  • 68 Bài viết


Bài này ảo nhỉ :))

Sử dụng khai triển Taylor với phần dư tích phân

$$f(x)=f(1)+f'(1)(x-1)+\int_1^x f''(t)(x-t)dt$$

$$\Rightarrow f(0)=f(1)-f'(1)+\int_0^1 tf''(t)dt \;\;, f(2)=f(1)+f'(1)+\int_1^2 f''(t)(2-t)dt$$

$$\Rightarrow f(0)+f(2)=2f(1)+\int_0^1 tf''(t)dt+\int_1^2(2-t)f''(t)dt$$

$$\Rightarrow 1 \le \sqrt{\int_0^1 t^2 dt \int_0^1 [f''(t)]^2 dt}+\sqrt{\int_1^2 (2-t)^2dt \int_1^2 [f''(t)]^2dt}$$

$$\Leftrightarrow 1 \le \dfrac{1}{\sqrt{3}} \sqrt{\int_0^1 [f''(x)]^2dx}+\dfrac{1}{\sqrt{3}} \sqrt{\int_1^2 [f''(x)]^2dx}$$

$$\Rightarrow \sqrt{3} \le \sqrt{2(\int_0^1 [f''(x)]^2dx+\int_1^2 [f''(x)]^2dx)}$$

$$\Leftrightarrow \int_0^2 [f''(x)]^2dx \ge \dfrac{3}{2}$$

Hic, cực nhọc quá :(

Một bài toán tương tự:

 

[Đặng Thành Nam]: Cho $f$ là hàm khả vi liên tục đến cấp hai trên $\left[ {0,2} \right]$ thỏa mãn $f(0) - 4f(1) + 3f(2) = 1$ và $f'(2) = 0$. Chứng minh rằng

$$\int\limits_0^2 {{{\left( {f''(x)} \right)}^2}dx}  \ge \frac{1}{{16}}$$


Bài viết đã được chỉnh sửa nội dung bởi dangnamneu: 31-03-2013 - 10:00

Giáo viên môn Toán tại website : http://vted.vn


#14
slbadguy

slbadguy

    Hạ sĩ

  • Thành viên
  • 76 Bài viết

Hay :0 mà các bạn cho mình xin tài liệu bất đẳng thức trong tích phân nhé






1 người đang xem chủ đề

0 thành viên, 1 khách, 0 thành viên ẩn danh